Finding formulas for the variable "a" and summations

Click For Summary
The discussion focuses on clarifying the transition from step 5 to step 6 in a mathematical problem involving summations. Participants explore the relationship between the expression within brackets and arithmetic series, questioning the impact of starting the summation index at i = 0 versus i = 1. They reference the formula for the sum of integers, ∑ i = n(n+1)/2, and how changing the index affects the final result. The conversation highlights confusion over the summation limits and the interpretation of the columns in the equation. Ultimately, the key point revolves around understanding the implications of different starting points in summation on the overall outcome.
Robb
Messages
225
Reaction score
8

Homework Statement


Can someone explain how to get from step 5 to step 6. I'm not seeing the link. Thanks in advance!

Homework Equations

The Attempt at a Solution

 

Attachments

Physics news on Phys.org
Think about what the expression inside the brackets looks like, doesn't it remind you of an arithmetic series?
 
Does it relate to ∑ i = n(n+1)/2? Because the indexes of sigma begin at i =0 we use n-1 instead of n+1, in the right side of the equation, which would indicate i = 1 to n (for n+1, that is)? Except, adding from zero (i.e., 0+1+2+3+4, +...+n) and adding from n-1 (i.e. (n-1)+(n-2)+(n-3)+...+ 1), I get n+1 for the last column when the columns (of each respective summation) are summed.
 
Your response indicates that you are quite confused.
Your first question seems to be:
Robb said:
Does it relate to ∑ i = n(n+1)/2? Because the indexes of sigma begin at i =0 we use n-1 instead of n+1, in the right side of the equation, which would indicate i = 1 to n (for n+1, that is)?
So, you seem to think that starting at i = 0 , rather than i = 1 somehow affects the final result?

Is 1 + 2 + 3 + ... + n any different than 0 + 1 + 2 + 3 + ... + n ?

Except, adding from zero (i.e., 0+1+2+3+4, +...+n) and adding from n-1 (i.e. (n-1)+(n-2)+(n-3)+...+ 1), I get n+1 for the last column when the columns (of each respective summation) are summed.
What columns are you referring to? I see no columns.

It seems that you know that ## \ \displaystyle \sum_{i=1}^K i = \dfrac{K(K+1)}{2} \,.\ ## Right?

So, rather than summing to K, or to n for that matter, are you not summing from 1 to (n−1) ?
 
Last edited by a moderator:
Question: A clock's minute hand has length 4 and its hour hand has length 3. What is the distance between the tips at the moment when it is increasing most rapidly?(Putnam Exam Question) Answer: Making assumption that both the hands moves at constant angular velocities, the answer is ## \sqrt{7} .## But don't you think this assumption is somewhat doubtful and wrong?

Similar threads

  • · Replies 4 ·
Replies
4
Views
3K
  • · Replies 9 ·
Replies
9
Views
2K
Replies
4
Views
2K
  • · Replies 1 ·
Replies
1
Views
2K
  • · Replies 5 ·
Replies
5
Views
1K
Replies
2
Views
2K
  • · Replies 3 ·
Replies
3
Views
966
  • · Replies 2 ·
Replies
2
Views
2K
  • · Replies 1 ·
Replies
1
Views
1K
  • · Replies 4 ·
Replies
4
Views
1K